Family Medicine 2020 Review

¡Supera tus tareas y exámenes ahora con Quizwiz!

A 63-year-old male with a history of controlled hypertension and benign prostatic hyperplasia presents to your office after an emergency department visit for a closed, displaced left clavicle fracture. The injury occurred when he slipped on ice while shoveling snow. His orthopedist has recommended surgery and has requested a preoperative evaluation. The patient's blood pressure is 128/82 mm Hg and cardiovascular and pulmonary examinations are normal. You determine that his risk for a major adverse cardiac event is <1% for this nonurgent and low-risk surgical intervention. He does not have any chest pain or shortness of breath. He has pain with movement of his left arm and is wearing a sling. Which one of the following would be most appropriate at this point? A) No further testing B) An EKG C) Exercise stress testing D) A chest radiograph E) Echocardiography

- A - This asymptomatic patient is at low risk for a major adverse cardiac event (<1%) and requires no further testing prior to undergoing a low-risk, nonurgent surgical intervention. - A routine preoperative EKG does not provide any benefit for asymptomatic patients undergoing low-risk surgical procedures (level of evidence 2). - A preoperative EKG may be considered for asymptomatic patients without known coronary artery disease who are undergoing intermediate and high-risk surgeries (SOR B). - A preoperative EKG is reasonable for patients who have known heart disease, peripheral artery disease, or cerebrovascular disease who are undergoing intermediate and high-risk surgeries (SOR B). - Preoperative echocardiography is recommended for patients with known or suspected moderate- or high-degree valvular heart disease (SOR C).

A 15-year-old male presents for a sports preparticipation examination. His family history includes hypertension in his mother. There are no other concerns. He does not take any medications or supplements. An examination shows a BMI at the 75th percentile for his age. His weight is 59 kg (130 lb) and his blood pressure is 130/88 mm Hg. On repeat measurements 30 minutes apart, his blood pressure is 132/90 mm Hg and 132/88 mm Hg. The remainder of the examination, including vital signs, is normal. Which one of the following would you recommend at this time? A) Therapeutic lifestyle changes with close follow-up in 2-3 weeks B) Yearly blood pressure monitoring C) Atenolol (Tenormin), 25 mg daily D) Enalapril (Vasotec), 5 mg daily E) Hydrochlorothiazide, 12.5 mg daily

- A - This patient has stage 1 hypertension based on his average blood pressure of 131/88 mm Hg after three measurements. - He is asymptomatic so the initial recommendation is therapeutic lifestyle changes and close follow-up in 2-3 weeks (SOR C). - If his blood pressure remains elevated, an evaluation for secondary hypertension would be appropriate. - Medication would be appropriate for stage 1 hypertension with either evidence of end-organ disease or persistent hypertension after a trial of therapeutic lifestyle changes or progression to stage 2 hypertension (SOR C). - Thiazide diuretics or ACE inhibitors would be appropriate choices for initial treatment. -Blockers are no longer considered first-line treatment for hypertension in adolescents or children.

An otherwise asymptomatic 7-year-old male has a blood pressure above the 95th percentile for gender, age, and height on serial measurements. Which one of the following studies would be most appropriate at this time? A) Renin and aldosterone levels B) 24-hour urinary fractionated metanephrines and normetanephrines C) Renal ultrasonography D) Doppler ultrasonography of the renal arteries E) A sleep study

- Answer C - Renal parenchymal diseases such as glomerulonephritis, congenital abnormalities, and reflux nephropathy are the most common cause of hypertension in preadolescent children. Preadolescent children with hypertension should be evaluated for possible secondary causes and renal ultrasonography should be the first choice of imaging in this age group.

Wolf Parkinson White Syndrome (WPW) treatment A) Adenosine (Adenocard) B) Amiodarone (Cordarone) C) Diltiazem (Cardizem) D) Metoprolol E) Catheter ablation

- Catheter ablation is the most appropriate treatment for a patient with symptomatic Wolff-Parkinson-White syndrome (WPW). Catheter ablation has a very high immediate success rate (96%-98%). - The most significant risk associated with the procedure is permanent atrioventricular block, which occurs in approximately 0.4% of procedures. - Adenosine and amiodarone are used for the acute management of supraventricular tachycardia, but not for long-term management.

- TB testing for a foreign traveler per CDC guidelines

- The CDC recommends either a TB skin test or an interferon-gamma release assay prior to leaving the United States. If the test is negative, the individual should repeat the testing 8-10 weeks after returning. A chest radiograph in asymptomatic individuals or prophylactic treatment at any point is not recommended. Isoniazid and rifampin are options for treatment of latent TB.

According to the most recent American College of Cardiology/American Heart Association guidelines, hypertension is defined as a blood pressure reading greater than

- The latest ACA/AHA guidelines promote a radical change in the management of hypertension, which they now define as a blood pressure ≥ 130/80 mm Hg. - Elevated blood pressure is defined as a systolic pressure of 120-129 mm Hg and a diastolic pressure <80 mm Hg. - A blood pressure of 130-139/80-89 mm Hg is classified as stage 1 hypertension and a systolic pressure ≥ 140 mm Hg or a diastolic pressure ≥90 mm Hg is classified as stage 2 hypertension. - The 2017 ACC/AHA guidelines recommend drug therapy for all patients with an average blood pressure >130 mm Hg systolic or >80 mm Hg diastolic despite a trial of lifestyle modification. -The 2017 ACC/AHA guidelines recommend drug therapy for all patients with an average blood pressure >130 mm Hg systolic or >80 mm Hg diastolic despite a trial of lifestyle modification

If a lesion is noted on the chest radiograph, which one of the following characteristics would be most suspicious for malignancy? A) A diameter of 5 mm ) Concentric calcifications C) Doubling in size in less than 1 month D) A nonsolid "ground glass" appearance E) Smooth borders

- The risk of malignancy is also higher in spiculated lesions, in lesions with asymmetric calcification, and in lesions located in an upper lobe.8 In contrast, nodules with smooth borders and a central or concentric pattern of calcification are more likely to be benign (Figure 1). Nodules can be classified as solid or subsolid. Solid nodules are more common, but subsolid nodules have a higher likelihood of malignancy.12 Subsolid nodules can be further characterized into pure ground-glass or part-solid in nature. Part-solid nodules include a combination of ground-glass and solid components, the latter obscuring lung architecture. - The risk of malignancy rises with increasing nodule size (maximum diameter). Approximately 80% of nodules greater than 20 mm are malignant, whereas only 1% of nodules between 2 and 5 mm are malignant.7,8 Malignant solid nodules typically have a doubling time within 400 days; therefore, experts agree that solid solitary pulmonary nodules that remain the same size over a two-year period are likely to be benign.6,9-11 Longer duration follow-up is advisable for ground-glass nodules, which generally have a longer doubling time

A 58-year-old male with well controlled type 2 diabetes presents for follow-up of ongoing exertional chest pain that has been present for months with no significant change. He has a remote history of myocardial infarction and recently underwent cardiac catheterization, which showed non-obstructing coronary artery disease with a left ventricular ejection fraction of 60%. His current medications include aspirin, 81 mg; atorvastatin (Lipitor), 80 mg; lisinopril (Prinivil, Zestril), 40 mg; metoprolol succinate (Toprol-XL), 100 mg; and metformin (Glucophage), 1000 mg twice daily. Home blood pressure monitoring shows an average blood pressure of 142/92 mm Hg and a pulse rate of 58 beats/min. A physical examination, including a cardiovascular examination, is unremarkable. Which one of the following medications would be the best addition to his current regimen? A) Amlodipine (Norvasc), 2.5 mg daily B) Diltiazem extended release (Cardizem LA), 120 mg daily C) Isosorbide mononitrate extended release, 30 mg daily D) Nifedipine (Procardia), 30 mg 3 times daily E) Ranolazine (Ranexa), 500 mg twice daily

- This patient has a blood pressure that is elevated according to all major current guidelines. Amlodipine, a long-acting dihydropyridine calcium channel blocker (CCB), is the best pharmacologic choice because it will lower blood pressure and treat angina without the risk of heart block. - Short-acting CCBs such as non-extended-release nifedipine may cause reflex tachycardia and are not recommended. -Nondihydropyridine CCBs such as diltiazem would put this patient at risk for heart block because he is already taking long-acting metoprolol and his heart rate is in the 50s. -Long-acting nitrates and ranolazine are options to treat stable angina but would not be effective blood pressure medications.

A 63-year-old male sees you after carotid ultrasonography at a local health fair showed a 50% occlusion of his left proximal internal carotid artery. He has no significant past medical history and has never had a TIA or stroke. In addition to a healthy diet and exercise, you would recommend: A) no further treatment or follow-up B) observation, and repeat ultrasonography in 1 year C) statin therapy, and repeat ultrasonography in 1 year D) statin therapy and referral to a vascular surgeon for consideration of a carotid artery stent E) statin therapy and referral to a vascular surgeon for consideration of carotid endarterectomy

-Asymptomatic carotid artery disease is considered a coronary artery disease risk equivalent; therefore, statin therapy is indicated. -Repeating ultrasonography annually to monitor for progression of the disease and to guide intervention is also considered reasonable. -The guidelines also state that it is reasonable to consider carotid endarterectomy for asymptomatic patients with >70% stenosis if the risks of perioperative complications are low.

67. A 55-year-old patient with a history of alcoholism is admitted through the emergency department with acute pancreatitis. Which one of the following tests performed at the time of admission can best predict the severity of pancreatitis? A) Hematocrit B) C-reactive protein C) Serum amylase D) Serum lipase E) CT of the abdomen

-C-reactive protein -Interleukin-6 -Interleukin-8 -Phospholipase A2 -Procalcitonin -Trypsinogen activation peptide (within a few hours) Hematocrit, BUN, and creatinine levels are the most useful predictors of the severity of pancreatitis, reflecting the degree of intravascular volume depletion. C-reactive protein is often elevated, but it is not as useful as hematocrit for predicting severity. Serum amylase and lipase have no prognostic value. CT evidence of severe pancreatitis lags behind clinical and laboratory evidence, and early CT underestimates the severity of the acute process. Ref: Forsmark CE, Vege SS, Ransom criteria for acute pancreatitis severity At admission: Age in years > 55 years WBC count > 16000 cells/mm3 Blood glucose > 11.11 mmol/L (> 200 mg/dL) Serum AST > 250 IU/L Serum LDH > 350 IU/L Within 48 hours: Serum calcium < 2.0 mmol/L (< 8.0 mg/dL) Hematocrit fall > 10% Oxygen (hypoxemia PaO2 < 60 mmHg) BUN increased by 1.8 or more mmol/L (5 or more mg/dL) after IV fluid hydration Base deficit (negative base excess) > 4 mEq/L Sequestration of fluids > 6 L

1) FABER Test 2) Leg roll test 3)Femoroacetabular impingement

1) A positive flexion, abduction, external rotation (FABER) test that produces pain at the sacroiliac joint, lumbar spine, and posterior hip is associated with sacroiliac joint dysfunction. 2) The log roll test involves passive supine internal and external rotation of the hip. When this test is positive for pain it is associated with piriformis syndrome. 3) While femoroacetabular impingement may be associated with a positive FABER test, it would produce pain in the groin. Greater trochanteric pain syndrome results in lateral hip pain rather than posterior pain. Osteoarthritis is usually associated with a limited range of motion and groin pain.

A 69-year-old female presents to your office with a 5-day history of cough and low-grade fever. She has a past history of hypertension and obstructive sleep apnea. Her daughter brought her in this morning because of worsening symptoms. The patient's temperature is 37.4°C (99.3°F), her blood pressure is 110/74 mm Hg, her pulse rate is 88 beats/min, her respiratory rate is 36/min, and her oxygen saturation is 95% on room air. She is alert and oriented to person, place, and time. A CBC and basic metabolic panel are normal except for an elevated WBC count of 12,500/mm3 (N 4300-10,800). A chest radiograph shows a right lower lobe infiltrate. This patient has a higher risk of mortality and should be considered for inpatient treatment due to her A) female sex B) underlying hypertension C) respiratory rate D) elevated WBC count E) abnormal chest radiograph

30-day mortality for CURB-65 (CRB-65)

COPD steroid treatment A 60-year-old patient is admitted to the hospital for a COPD exacerbation. For this patient, which one of the following would be the most appropriate prednisone dosage? A) 40 mg daily for 5 days B) 40 mg daily for 10 days C) 60 mg daily, tapered over 6 days D) 60 mg daily for 10 day

A Short courses of systemic corticosteroids increase the time to subsequent exacerbation, decrease the rate of treatment failure, shorten hospital stays, and improve hypoxemia and forced expiratory volume in one second (FEV1).1,6,7,9,17-20 Administration of oral corticosteroids early in an exacerbation decreases the need for hospitalization.21 A randomized controlled trial (RCT) of patients with COPD compared eight weeks of corticosteroids, two weeks of corticosteroids, and placebo; participants in the treatment groups had fewer treatment failures than those in the control group.17 Treatment failure rates were the same for long and short courses of corticosteroids. High-dosage corticosteroid regimens (methylprednisolone [Solu-Medrol], 125 mg intravenously every six hours) and low-dosage regimens (prednisolone, 30 mg orally daily) decrease the length of hospitalization and improve FEV1 compared with placebo.17,19 [ corrected] An RCT comparing oral and intravenous prednisolone in equivalent dosages (60 mg daily) showed no difference in lengths of hospitalization and rates of early treatment failure. Because oral corticosteroids are bioavailable, inexpensive, and convenient, parenteral corticosteroids should be reserved for patients with poor intestinal absorption or comorbid conditions that prevent safe oral intake (e.g., decreased mental status, vomiting).5,6 Inhaled corticosteroids have no role in the management of an acute exacerbation.8

An 85-year-old female with a previous history of diabetes mellitus, hypertension, dementia, and peptic ulcer disease has been in a skilled nursing facility for 4 weeks for rehabilitation after a hip fracture repair secondary to a fall during an ischemic stroke. She is transported to the emergency department today when she develops confusion, shortness of breath, and diaphoresis. Her blood pressure is 172/98 mm Hg, her heart rate is 122 beats/min with an irregular rhythm, and her respiratory rate is 22/min. An EKG demonstrates atrial fibrillation and 0.2 mV ST-segment elevation compared to previous EKGs. Her first troponin level is elevated. Which one of the following conditions in this patient is considered an ABSOLUTE contraindication to fibrinolytic therapy? A) Poorly controlled hypertension B) Peptic ulcer disease C) Alzheimer's dementia D) Hip fracture repair E) Ischemic stroke

A history of an ischemic stroke within the past 3 months is an absolute contraindication to fibrinolytic therapy in patients with an ST-elevation myocardial infarction (STEMI), unless the stroke is diagnosed within 4½ hours. Poorly controlled hypertension, dementia, peptic ulcer disease, and major surgery less than 3 weeks before the STEMI are relative contraindications that should be considered on an individual basis.

An otherwise healthy 42-year-old male presents to your office with low back pain that started a week ago after he lifted a heavy box. Since the time of his injury he has been having consistent pain, numbness, and tingling that radiates down the back of his right leg to his calf. Which one of the following would you order at this time? A) No imaging B) Plain radiography C) CT D) MRI

ANSWER: A Uncomplicated acute low back pain and/or radiculopathy is a benign, self-limited condition and early imaging is associated with worse overall outcomes and is likely to identify minor abnormalities even in asymptomatic patients. Imaging for acute low back pain should be reserved for cases that are suspicious for cauda equina syndrome, malignancy, fracture, or infection. In the absence of red flags such as progressive motor or sensory loss, new urinary retention or overflow incontinence, a history of cancer, a recent invasive spinal procedure, or significant trauma relative to age, imaging is not warranted regardless of whether radiculopathy is present, unless symptoms persist despite a trial of at least 6 weeks of medical management and physical therapy.

A healthy 55-year-old white male with a family history of coronary artery disease sees you for a routine health maintenance visit. He asks you what he could do to decrease his risk of cardiovascular disease. He is a nonsmoker, does not drink alcohol, and has no history of substance abuse. His BMI is normal and the physical examination is otherwise unremarkable. His vital signs include a heart rate of 80 beats/min, a blood pressure of 119/70 mm Hg, a respiratory rate of 15/min, and a temperature of 37.0°C (98.6°F). Laboratory Findings Fasting glucose....................................................92 mg/dL Total cholesterol..................................................190 mg/dL LDL-cholesterol...................................................98 mg/dL HDL-cholesterol ..........................................................50 mg/dL Triglycerides .................................................................. 145 mg/dL His calculated 10-year risk for cardiovascular disease is 5.4%. Which one of the following has the best evidence to prevent cardiovascular disease in a patient such as this? A) Moderate-intensity exercise, 150 minutes weekly B) A low-dose statin C) Aspirin, 81 mg daily D) Fish oil supplements E) Niacin supplements

ANSWER: A A systematic evidence review released by the U.S. Preventive Services Task Force (USPSTF) noted that the most active people had median cardiovascular risk reductions of about 30%-35% when compared with the least active. Statins are beneficial for both primary and secondary prevention of cardiovascular disease, but the benefit is greater when the baseline risk is greater. Current guidelines would not support statin therapy for a patient with a 10-year risk of atherosclerotic cardiovascular disease (ASCVD) <5%. Fish oil supplements have not proven to be useful for primary prevention of ASCVD. Aspirin is recommended for the prevention of cardiovascular disease in adults 50-59 years of age with a >10% 10-year ASCVD risk who are not at increased risk of bleeding, are expected to live at least 10 years, and are willing to take low-dose daily aspirin for 10 years (USPSTF B recommendation). Niacin is no longer recommended for cardiovascular risk reduction due to a lack of evidence for benefit.

A 75-year-old male with a history of hypertension, TIA, and atrial fibrillation sees you for follow-up. Ten days ago he was on vacation in another state when he developed chest pain. He went to a local hospital where he was diagnosed with an ST-elevation myocardial infarction (STEMI) and was taken immediately for cardiac catheterization. He had a drug-eluting stent placed in his left anterior descending artery. He brings some discharge paperwork with him, including a medication list, but has not yet seen a local cardiologist. He is concerned that he is taking too many blood thinners. He feels well and does not have any chest pain, shortness of breath, or excessive bleeding or bruising. Prior to his STEMI the patient was taking lisinopril (Prinivil, Zestril), 10 mg daily; warfarin (Coumadin), 2.5 mg daily; and metoprolol succinate (Toprol-XL), 25 mg daily. Upon discharge he was instructed to continue all of those medications and to add clopidogrel (Plavix), 75 mg daily, and aspirin, 81 mg daily. The patient's vital signs and physical examination are normal except for an irregularly irregular rhythm on the cardiovascular examination. His INR is 2.5. Which one of the following would be most appropriate at this time? A) Continue the current regimen B) Discontinue aspirin C) Discontinue clopidogrel D) Discontinue warfarin E) Decrease warfarin with a goal INR of 1.5-2.0

ANSWER: A Current guidelines recommend that patients with an ST-elevation myocardial infarction (STEMI) who also have atrial fibrillation take dual antiplatelet therapy such as aspirin plus clopidogrel and a vitamin K antagonist, with a goal INR of 2.0-3.0. If a patient was already taking a direct-acting oral anticoagulant (DOAC) instead of warfarin for atrial fibrillation, the patient should continue with the DOAC in addition to dual antiplatelet therapy. The duration of triple therapy should be as short as possible, and aspirin can often be discontinued after 1-3 months. However, this patient's STEMI occurred less than 2 weeks ago and he should continue triple therapy.

A 58-year-old male sees you for follow-up of diabetic gastroparesis. He has tried managing his symptoms with more frequent meals and taking in more calories in semisolid or liquid form. These approaches have been unsuccessful in controlling his symptoms and he would like to try a medication. Which one of the following would be considered first-line pharmacotherapy for this patient? A) Metoclopramide (Reglan) B) Nortriptyline (Pamelor) C) Omeprazole (Prilosec) D) Ondansetron (Zofran) E) Ranitidine (Zantac)

ANSWER: A Diabetic gastroparesis is a delay in the emptying of food from the upper gastrointestinal tract in the absence of a mechanical obstruction of the stomach or duodenum. Metoclopramide is the only prokinetic agent that has been studied specifically for long-term use in gastroparesis and is considered first-line therapy (SOR B). It is among the only FDA-approved medications for gastroparesis. Nortriptyline is a prokinetic agent but has not been shown to be more effective than placebo for decreasing gastroparesis symptoms. Proton pump inhibitors such as omeprazole, histamine H2-receptor antagonists such as ranitidine, and ondansetron delay gastric emptying and should be withheld in patients with gastroparesis whenever possible.

A 17-year-old male presents with right lateral ankle pain after twisting his ankle in a basketball game earlier today. The area is mildly swollen. A radiograph shows soft-tissue edema but no evidence of fracture. Which one of the following would be the most appropriate management of this condition? A) Use of a functional ankle brace with early mobilization B) Prolonged use of a patellar tendon strap to help unload the ankle joint C) Immobilization with compression taping for a minimum of 2 weeks D) Referral to physical therapy for neuromuscular retraining

ANSWER: A Early mobilization leads to better outcomes with ankle sprains, and using a functional ankle brace such as a semi-rigid air stirrup brace or a soft lace-up ankle brace will protect the ankle from inversion and eversion sprains while still allowing for mobility during physical activity. These braces lead to improved functional outcomes at 1 month when compared to elastic compression taping. Patellar tendon straps relieve the pain associated with patellar tendinopathy but are not indicated with acute ankle sprains. Use of a semi-rigid air stirrup splint has been shown to be superior to neuromuscular training for prevention of recurrent ankle sprains.

A 56-year-old male with a history of hepatitis C cirrhosis is admitted to the hospital with gastrointestinal (GI) bleeding. The patient has been stable, taking only furosemide (Lasix) and spironolactone (Aldactone). Upper GI endoscopy confirms variceal bleeding and the gastroenterologist performs appropriate variceal banding. A nurse calls you because laboratory studies that were ordered in the emergency department reveal a serum ammonia level of 120 g/dL (N 39-90). The patient has no signs of confusion, insomnia, or decreased mental alertness. A physical examination reveals mild ascites but no other abnormalities. Which one of the following would be most appropriate for addressing the elevated ammonia level? A) No additional treatment B) Lactulose (Kristalose) C) Methotrexate D) Neomycin E) Prednisone

ANSWER: A Elevated ammonia levels may occur with urea cycle disorders, portosystemic shunting, urinary tract infection from urease-producing organisms, gastrointestinal bleeding, shock, renal disease, heavy exercise, smoking, parenteral nutrition, salicylate intoxication, use of medications including diuretics, and alcohol use. In patients with chronic liver disease, hepatic encephalopathy is diagnosed based on the overall clinical presentation and not on ammonia levels. A normal ammonia level does not exclude the diagnosis of hepatic encephalopathy, nor does an elevated ammonia level establish the diagnosis. This patient had an elevated serum ammonia level that was found incidentally during his hospital admission for gastrointestinal bleeding. Because there was no clinically significant encephalopathy, treatment based on ammonia levels is not indicated. The patient's elevated ammonia level was probably from diuretic use. Lactulose, methotrexate, neomycin, or prednisone would not be appropriate.

An 80-year-old former smoker sees you for a 6-month follow-up for hypertension. He is taking carvedilol (Coreg), amlodipine (Norvasc), and low-dose aspirin. His home blood pressure readings have been 130-150/80-90 mm Hg. Over the last 4 months he has developed pain in his thighs when walking to his mailbox a block away. The pain resolves after he sits for a few minutes. On examination he has a blood pressure of 135/85 mm Hg, a heart rate of 72 beats/min, a BMI of 26 kg/m2 , and an oxygen saturation of 95% on room air. Examinations of the heart and lungs are normal. There is dependent rubor of both legs but posterior tibial pulses are palpable. No ulcerations are noted. You obtain ankle-brachial indices of 0.85 on the left and 0.80 on the right. You prescribe a daily walking program. Which one of the following additional measures would be most appropriate for this patient? A) Add atorvastatin (Lipitor) B) Add clopidogrel (Plavix) C) Add lisinopril (Prinivil, Zestril) to achieve a goal blood pressure <120/80 mm Hg D) Discontinue aspirin and start warfarin (Coumadin) E) Refer to a vascular surgeon

ANSWER: A Management of asymptomatic peripheral artery disease (PAD) should initially be conservative and should include a walking program (SOR A), smoking cessation, and a healthy diet. Statins should be started for all patients with PAD regardless of their LDL-cholesterol levels (SOR A). High-intensity statins should be used if tolerated. A single antiplatelet agent is recommended for patients with PAD. Both aspirin and clopidogrel are effective in the reduction of stroke, but the combination of the two is recommended only after revascularization surgery. Blood pressure control is indicated in patients with PAD but no antihypertensive class is clearly superior to another, although there is some evidence that ACE inhibitors may have additional benefits in terms of walking and pain. In an 80-year-old patient, lowering blood pressure below 120/80 mm Hg can be associated with significant side effects, including a greater risk of falls. Anticoagulants have not been shown to reduce the risk of major cardiovascular events in patients with PAD and they increase the risk of life-threatening bleeding. Referral to a vascular surgeon or for angiography is indicated if conservative therapy fails or symptoms worsen acutely, pain occurs at rest, or the patient develops ulcerations or loss of tissue.

. A 74-year-old female with a long-standing history of coronary artery disease is hospitalized for pneumonia. The patient improves with treatment and is hemodynamically stable. An EKG performed on the third day of hospitalization is shown below. Which one of the following would be the most appropriate next step? A) Cardiac rhythm monitoring with no additional treatment B) Atropine C) Transcutaneous pacing D) Transvenous pacing

ANSWER: A Second degree Mobitz type I (Wenckebach) heart block is characterized by an intermittent blockade of electrical impulses from the atria to the ventricles at the level of the atrioventricular node. This prevents generation of a QRS complex. It is characterized by progressive prolongation of the PR interval until a P wave is not followed by a QRS complex. P waves come at regular intervals so PP intervals are normal. Following the missed QRS complex, the PR interval returns to its baseline duration. A pacemaker is not recommended in patients with second degree Mobitz type I heart block who are asymptomatic. It is recommended in symptomatic patients, however, and is guided by electrophysiologic studies.

A 52-year-old male sees you for follow-up after his third episode of acute gout in the last year. The patient has otherwise been in good health with recent normal laboratory results, including normal renal function. You decide to begin therapy with allopurinol (Zyloprim). To reduce the risk of recurrent gout episodes, you also initiate therapy with which one of the following for the next 6 months? A) Colchicine (Colcrys) B) Methotrexate C) Prednisone D) Probenecid

ANSWER: A Strong evidence suggests that prophylaxis with either colchicine or NSAIDs reduces the risk for acute gout attacks in patients starting urate-lowering therapy. The optimal duration of such prophylactic therapy is unknown, but moderate evidence suggests that it should be longer than 8 weeks. Although prednisone would be helpful it is not the preferred agent in this patient with normal renal function. Prophylaxis with aspirin, methotrexate, or probenecid is also not appropriate.

An 80-year-old male sees you for the first time. He is asymptomatic except for some fatigue. His pulse rate is 50 beats/min. An EKG shows a prolonged PR interval. Which one of the following medications in his current regimen is the most likely explanation for these findings? A) Donepezil (Aricept) B) Escitalopram (Lexapro) C) Lisinopril (Prinivil, Zestril) D) Memantine (Namenda) E) Zolpidem (Ambien)

ANSWER: A The 2015 American Geriatrics Society Beers Criteria for potentially inappropriate medication use in older adults 65 years of age states that donepezil use should be avoided in patients with syncope, due to an increased risk of bradycardia (Moderate Evidence Level; Strong Strength of Recommendation). Donepezil is a cholinesterase inhibitor. Due to their cholinergic effect, these medications have a vagotonic effect on the sinoatrial and atrioventricular nodes. This can cause bradycardia or heart block in patients with or without underlying cardiac conduction abnormalities. Syncope has been reported with these medications. 83 Memantine is an N-methyl-D-aspartate receptor antagonist and is not associated with bradycardia. Escitalopram, lisinopril, and zolpidem are also not associated with bradycardia.

A 13-year-old female is brought to your office by her father for evaluation of left knee pain. It has been slowly worsening over the past several weeks and she does not recall any inciting injury or event. She plays soccer and participates in practice 3 days a week with games on the weekends. She is finding it increasingly difficult to participate fully due to pain. She notes some swelling and tenderness just below her kneecap. She has tried icing the area after activity and has recently started taking ibuprofen with moderate relief of the pain. On examination you note prominence of the tibial tubercle with tenderness to palpation. The remainder of the knee examination is unremarkable. Which one of the following is indicated at this time to further assess this condition? A) No imaging B) Radiography C) Ultrasonography D) CT E) MRI

ANSWER: A This patient has Osgood-Schlatter disease, a common cause of knee pain in active children with immature skeletons. It occurs as a result of abnormal development, injury, or overuse of the growth plate and the surrounding ossification centers. Osteochondrosis is a more general term for this condition, which can occur at growth plates around other joints, including the hip, foot, elbow, and back. In Osgood-Schlatter disease repetitive traction of the patellar tendon on the tibial tubercle ossification center leads to inflammation and pain. Imaging is not required to make the diagnosis when patients present with typical symptoms and physical examination findings. Radiographs may be obtained if there is uncertainty about the diagnosis. Radiographic findings in Osgood-Schlatter disease include soft-tissue swelling and fragmentation of the tibial tubercle. This condition is self-limited and treatment consists of activity modification and the use of acetaminophen or NSAIDs. An orthopedic referral is indicated if symptoms persist in a patient with a mature skeleton.

A 2-year-old male is brought to your office by his mother. The child has a 2-day history of a runny nose and mild cough associated with a subjective fever. The cough worsened last night. The patient has had a reduced appetite but a good intake of fluids. On examination the child has an axillary temperature of 37.4°C (99.3°F), a heart rate of 120 beats/min, a respiratory rate of 26/min, a weight of 16 kg (35 lb), and an oxygen saturation of 96% on room air. He appears mildly ill but is alert and does not show any signs of distress, and has a prominent high-pitched barking cough. You note that he has clear rhinorrhea, the tympanic membranes are normal, and the oropharynx is moist and clear. Auscultation reveals inspiratory stridor, but there are no signs of respiratory distress. The patient's skin has good turgor with no rash. Which one of the following would be the most appropriate next step in the management of this child? A) Administration of dexamethasone, 0.6 mg/kg orally in a single dose B) Initiation of oral amoxicillin, 40 mg/kg twice daily C) Administration of albuterol, 2.5 mg/3 mL via nebulizer D) Administration of racemic epinephrine 2.25% solution (Asthmanefrin), 0.5 mL via nebulizer E) Transfer to a hospital emergency department for stabilization and hospital admission

ANSWER: A This patient has mild croup based on the clinical findings. A single dose of dexamethasone is recommended in all cases of croup (SOR A). Hospitalization is not necessary if the child is stable. Racemic epinephrine, which has been shown to reduce symptoms at 30 minutes but not at 2 hours or 6 hours, is recommended for the treatment of moderate to severe croup when patients are being observed in a medical setting such as the emergency department or hospital (SOR A). Amoxicillin and albuterol are not indicated in the management of croup.

A 59-year-old plumber presents with swelling of his left elbow. An examination reveals swelling over the olecranon but no erythema or warmth. His uric acid levels are normal and he has no history of inflammatory disease. He has not had a fever. The swollen area is not painful and he has a normal range of motion. Which one of the following would be the most appropriate next step? A) Padding, ice, and elevation B) Empiric antibiotics C) Bursal aspiration D) Plain radiographs E) Orthopedic referral

ANSWER: A This patient has olecranon bursitis, which is a superficial bursitis caused by chronic microtraumas to the affected area. The initial management for this condition includes conservative measures such as padding, elevation, icing, and analgesics (SOR B). If significant pain is associated with the swelling, or a decrease in range of motion is present due to severe swelling, aspiration should be offered. This is not indicated in this particular case and should be avoided to reduce the risk of septic bursitis. Septic bursitis would require empiric antibiotics to cover common skin organisms (SOR B). Aspiration should be performed if infection is suspected and the aspirate should be sent for a cell count, Gram stain, culture, glucose measurement, and crystal analysis (SOR C). Plain radiographs are indicated only if there is acute trauma and concern that a fracture may be present. If recurrent superficial bursitis occurs, a referral for surgery is indicated.

A 34-year-old male with sickle cell disease has a new onset of mild to moderate thirst and polyuria. He ate a large meal about 2 hours ago. An examination reveals a BMI of 32 kg/m2 . Results of a urinalysis performed by your staff include 3+ glucose and no ketones. His blood glucose level is 288 mg/dL and his hemoglobin A1c is 5.2%. Which one of the following would be most appropriate at this point to help diagnose and monitor this patient's glycemic control? A) A serum fructosamine level B) A repeat hemoglobin A1c C) A 2-hour glucose tolerance test D) Hemoglobin electrophoresis E) Referral to an endocrinologist

ANSWER: A This patient with sickle cell disease has a new onset of diabetes mellitus. Hemoglobinopathies falsely lower hemoglobin A1c as a result of hemolysis and abnormal glycation. Fructosamine correlates well with hemoglobin A1c levels and is recommended instead of hemoglobin A1c for monitoring glucose control in patients with diabetes and hemoglobinopathies. A 2-hour glucose tolerance test or hemoglobin electrophoresis would not provide useful information. Referral to an endocrinologist is not indicated at this point because the patient has not failed primary care management.

A 44-year-old female presents for a pretravel consultation and asks about medication options for traveler's diarrhea. She will be on an organized tour traveling to a country with a very low risk for this problem. She plans to take all precautions to further reduce her risk but would also like you to recommend a medication she can take. Which one of the following would be an appropriate recommendation? A) A short course of azithromycin (Zithromax) if she develops diarrhea B) Loperamide (Imodium) daily, starting 1 day prior to travel and continued until 1 day after returning home C) Probiotics daily, starting 1 week prior to travel and continued until 1 week after returning home D) Ciprofloxacin (Cipro) daily, starting 2 weeks prior to travel and continued until 4 weeks after returning home E) Bismuth subsalicylate daily, starting 2 weeks prior to travel and continued until 4 weeks after returning home

ANSWER: A Traveler's diarrhea is the most common infection in international travelers. A short course of antibiotics can be taken after a traveler develops diarrhea and usually shortens the duration of symptoms (SOR A). Azithromycin is preferred to treat severe traveler's diarrhea. Rifaximin or fluoroquinolones may be used to treat severe nondysenteric traveler's diarrhea. Prophylactic antibiotics are not routinely recommended.

A 42-year-old male with hypertension and hyperlipidemia sees you for a routine health maintenance examination. His blood pressure is 185/105 mm Hg. He does not have any current symptoms, including headache, chest pain, edema, or shortness of breath. He is adherent to his current medication regimen, which includes lisinopril (Prinivil, Zestril), 10 mg daily, and simvastatin (Zocor), 20 mg at night. A thorough history and physical examination are both unremarkable. Which one of the following would be the most appropriate next step? A) A 30-minute rest period followed by a repeat blood pressure reading B) Clonidine (Catapres), 0.2 mg given in the office C) A comprehensive metabolic panel, fasting lipid profile, and TSH level D) A stress test E) Hospital admission for blood pressure reduction

ANSWER: A The first step in the management of severe hypertension is determining whether a hypertensive emergency is present. A thorough history and physical examination are crucial (SOR C). Severe hypertension (blood pressure >180 mm Hg systolic or >110 mm Hg diastolic) with end-organ damage constitutes a hypertensive emergency. A physical examination should center on evaluating for papilledema, neurologic deficits, respiratory compromise, and chest pain. If end-organ damage is present the patient should be hospitalized for monitored blood pressure reduction and further diagnostic workup. If end-organ damage is not present and the physical examination is otherwise normal, a 30-minute rest with reevaluation is indicated. Approximately 30% of patients will improve to an acceptable blood pressure without treatment (SOR C). Home medications should then be adjusted with outpatient follow-up and home blood pressure monitoring (SOR A). Short-acting antihypertensives are indicated if mild symptoms are noted such as headache, lightheadedness, nausea, shortness of breath, palpitations, anxiety, or epistaxis. Diagnostic testing is not immediately indicated for asymptomatic patients (SOR C). A basic metabolic panel or other testing should be considered if mild symptoms are present. Aggressive lowering of blood pressure can be detrimental and a gradual reduction over days to weeks is preferred (SOR C).

A 54-year-old male develops chest pain while running. He is rushed to the emergency department of a hospital equipped for percutaneous coronary intervention. An EKG shows 3 mm of ST elevation in the anterior leads. He is diaphoretic and cool with ongoing chest pain. His blood pressure is 80/50 mm Hg, his pulse rate is 116 beats/min, and his oxygen saturation is 98% on room air. You would immediately administer: A) a -blocker B) dual antiplatelet therapy and an anticoagulant C) intravenous fibrinolytic therapy D) an intravenous vasopressor

ANSWER: B - This patient is likely experiencing an acute anterior wall myocardial infarction with possible incipient cardiogenic shock. - Along with initiating the hospital's protocol for myocardial infarction, immediate treatment should include dual antiplatelet therapy with a 325-mg dose of nonenteric aspirin, a P2Y12 inhibitor (clopidogrel, prasugrel, or ticagrelor), and an anticoagulant (unfractionated heparin or bivalirudin). - Given the possibility of cardiogenic shock, -blockers should not be used. Unless more than a 2-hour delay in percutaneous coronary intervention is expected, fibrinolytics should not be administered.

A 55-year-old male sees you for an annual health maintenance visit. He is a former smoker and has a history of type 2 diabetes mellitus, hypertension, and hyperlipidemia. He had a normal colonoscopy at age 50, and had an ST-elevation myocardial infarction 2 years ago treated with a drug-eluting stent. He is currently asymptomatic and does not have any chest pain, hypoglycemia, dyspepsia, melena, or rectal bleeding. His medications include metformin (Glucophage), 2000 mg daily; glimepiride (Amaryl), 2 mg daily; bisoprolol (Zebeta), 5 mg daily; losartan/hydrochlorothiazide (Hyzaar), 50 mg/12.5 mg daily; rosuvastatin (Crestor), 20 mg daily; clopidogrel (Plavix), 75 mg daily; and aspirin, 81 mg daily. His blood pressure is 128/76 mm Hg and his heart rate is 63 beats/min. A physical examination is unremarkable. His hemoglobin A1c is 6.4%. You recommend that the patient stop taking A) aspirin B) clopidogrel C) aspirin and clopidogrel D) metformin

ANSWER: B Patients with drug-eluting stents should be on dual antiplatelet therapy with aspirin plus a thienopyridine such as clopidogrel for a minimum of 1 year. At the time of this patient's visit, 2 years after the stent placement, there is no indication to continue clopidogrel, but aspirin therapy should be continued indefinitely. All of the patient's other medications have current active indications and should be continued, although if the patient experiences hypoglycemia, the sulfonylurea could be decreased or discontinued.

A gravida 2 para 0 at 34 weeks gestation presents to your office because of diffuse itching. She does not have any known allergies other than seasonal allergies, and she does not have any new contacts. An examination is normal other than some scattered excoriations, and there is no other distinct rash. She has tried moisturizers but her symptoms have not improved. Which one of the following would be most appropriate at this point? A) Monitoring for the development of a rash B) Liver function tests and serum bile acid levels C) Topical corticosteroids D) Oral antihistamines E) Varicella-zoster immune globulin

ANSWER: B Whenever a pregnant woman presents with pruritus without a primary rash, it is important to evaluate her for intrahepatic cholestasis of pregnancy. This diagnosis is associated with increased fetal mortality and warrants increased antenatal surveillance as well as possible induction by 35-37 weeks gestation. It is most appropriate to check for elevation of liver function tests and serum bile acids. Emollients, topical corticosteroids, and oral antihistamines can all be helpful for pruritus and certain rashes, but in this patient it is most important to promptly look for the cause of the pruritus. Varicella-zoster immune globulin would be indicated if she had no immunity to varicella and had been exposed to varicella or if she had a rash that was suspected to be chickenpox.

A 54-year-old male comes to your office to establish care. He has a past history of hypertension treated with lisinopril (Prinivil, Zestril) and hydrochlorothiazide but has not taken his medications for over a year. He does not have any symptoms, including chest pain, shortness of breath, or headache. On examination his blood pressure is 200/115 mm Hg on two separate readings taken 5 minutes apart. The remainder of the physical examination is normal. Which one of the following management options would be most appropriate? A) Institute out-of-office monitoring with an ambulatory device and follow up in 2 weeks B) Restart the patient's previous antihypertensive medications and follow up within 1 week C) Administer a short-acting antihypertensive medication in the office to lower his blood pressure to <160/100 mm Hg D) Hospitalize for hypertensive emergency

ANSWER: B This patient has severe asymptomatic hypertension (systolic blood pressure 180 mm Hg or diastolic blood pressure 110 mm Hg). If there were signs or symptoms of acute target organ injury, such as neurologic deficits, altered mental status, chest pain, shortness of breath, or oliguria, hospitalization for a hypertensive emergency would be indicated. Because this patient was asymptomatic and has a known history of hypertension, restarting his prior antihypertensive regimen and following up in 2 weeks would be the most appropriate management option. If he had no past history of hypertension it would be reasonable to consider out-of-office monitoring with an ambulatory device for 2 weeks before initiating treatment. In the absence of acute target organ injury, blood pressure should be gradually lowered to less than 160/100 mm Hg over several days to weeks. Aggressively lowering blood pressure can lead to adverse events such as myocardial infarction, cerebrovascular accident, or syncope, so administering a short-acting antihypertensive medication in the office should be reserved for the management of hypertensive emergencies.

A 67-year-old male presents to your office because of fatigue and a syncopal episode. His vital signs in the office are normal. An examination reveals a harsh systolic murmur best heard over the second right intercostal space radiating to the neck. Echocardiography confirms your suspected diagnosis. Which one of the following is the only treatment that improves mortality with this condition? A) -Blockers B) Antimicrobial prophylaxis for bacterial endocarditis C) Aortic valve replacement D) Mitral valve repair E) Ventricular septal defect closure

ANSWER: C - This patient has symptomatic severe aortic stenosis. The only treatment that improves this condition is aortic valve replacement (SOR B). Transcutaneous aortic valve replacement may be an alternative for patients who are not candidates for surgery. - Blockers must be used with caution due to the risk of depressing left ventricular systolic function. They have not been shown to improve mortality. Symptomatic mitral valve regurgitation may require mitral valve intervention. However, these murmurs are holosystolic, high pitched, and best heard at the cardiac apex. A ventricular septal defect can cause a loud holosystolic murmur with an associated thrill heard best at the third/fourth interspace along the sternal border.

An adult male sees you for follow-up of metabolic syndrome associated with type 2 diabetes and hyperlipidemia. Routine laboratory findings include an AST level of 73 U/L (N 10-59) and an ALT level of 62 U/L (N 13-40). These levels are persistently elevated 3 months later. He states that he drinks 1-2 glasses of wine per week and does not use acetaminophen. His medications include metformin (Glucophage) and atorvastatin (Lipitor). Which one of the following would be most appropriate at this point? A) Discontinue atorvastatin B) Discontinue metformin C) Screen for hepatitis and hemochromatosis D) Order CT of the abdomen E) Refer for a liver biopsy

ANSWER: C This patient most likely has nonalcoholic fatty liver disease (NAFLD). The initial evaluation should include studies to rule out uncommon, but not rare, causes of liver disease. This would include viral hepatitis studies, iron studies for hemochromatosis, serum albumin levels, and a CBC. Discontinuing statin therapy is generally not necessary because statins have been shown to be safe for patients with chronic liver disease. Metformin is not commonly associated with elevated transaminases. If NAFLD is determined to be the most likely cause, an NAFLD fibrosis score should be calculated to preclude the need for a liver biopsy.

A 67-year-old male is admitted to the hospital for community-acquired pneumonia. An examination reveals a temperature of 40.0°C (104.0°F), a respiratory rate of 50/min, a pulse rate of 110 beats/min, a blood pressure of 90/50 mm Hg, and an oxygen saturation of 88% on room air. The patient is confused and requires aggressive fluid resuscitation for hypotension and he is transferred to the intensive-care unit. He has no known additional risk factors or exposures. In addition to treatment with ceftriaxone and azithromycin (Zithromax), which one of the following medications is most likely to result in improved outcomes? A) Clindamycin (Cleocin) B) Levofloxacin (Levaquin) C) Methylprednisolone (Medrol) D) Oseltamivir (Tamiflu)

ANSWER: C This patient has severe community-acquired pneumonia based on clinical criteria, including an elevated respiratory rate, confusion, and hypotension requiring aggressive fluid resuscitation. Corticosteroids such as methylprednisolone have been shown to improve clinical outcomes such as length of stay, duration of antibiotic treatment, and the risk of developing adult respiratory distress syndrome. The preferred choice of antibiotic treatment for patients in the intensive-care unit is a -lactam antibiotic (ceftriaxone, cefotaxime) or ampicillin/sulbactam, plus a macrolide alone or a macrolide and a respiratory fluoroquinolone. The addition of levofloxacin is not necessarily preferred over just ceftriaxone and azithromycin. Clindamycin is not indicated in the absence of risk factors for anaerobic infection such as aspiration or alcoholism. Oseltamivir is not indicated in the absence of known or suspected influenza infection.

A 52-year-old female sees you because of a vaginal discharge. An examination reveals a malodorous, greenish-yellow, frothy discharge, and inflammation of the cervix and vagina. Which one of the following is the most likely diagnosis? A) Atrophic vaginitis B) Irritant/allergic vaginitis C) Bacterial vaginosis D) Trichomoniasis E) Vulvovaginal candidiasis

ANSWER: D Trichomoniasis classically presents as a greenish-yellow, frothy discharge with a foul odor. Erythema and inflammation of the vagina and cervix are often present and can include punctate hemorrhages (strawberry cervix). Atrophic vaginitis may cause a thin, clear discharge and is usually associated with a thin, friable vaginal mucosa. Irritant/allergic vaginitis causes burning and soreness with vulvar erythema but usually does not cause any significant discharge. Bacterial vaginosis more commonly presents as a thin, homogenous discharge with a fishy odor and no cervical or vaginal inflammation. Vulvovaginal candidiasis presents with white, thick, cheesy, or curdy discharge.

Which one of the following is an indication for considering atrial ventricular nodal ablation in a patient with atrial fibrillation? A) Hemodynamic instability B) Successful cardioversion to sinus rhythm C) Inability of the patient to be anticoagulated D) Atrial fibrillation refractory to medical therapy E) Persistent atrial fibrillation in a patient who has been successfully rate controlled and anticoagulated for several years

ANSWER: D Atrial ventricular nodal ablation is recommended for patients whose atrial fibrillation is refractory to medical therapy, and requires that patients be anticoagulated for at least 1 month prior to the procedure and for several months afterward (SOR C). Patients with atrial fibrillation who are hemodynamically unstable should be considered for emergent cardioversion (SOR C). Atrial ventricular nodal ablation is not necessary for patients successfully converted to sinus rhythm or for those who are successfully treated with medical interventions for rate control and anticoagulation.

You see a 58-year-old female who received a drug-eluting stent 10 days ago during a hospitalization for acute coronary syndrome and coronary artery disease. She asks for recommendations about anticoagulation. You determine that she is not at high risk for bleeding. Which one of the following would you recommend? A) Long-term aspirin use B) Clopidogrel (Plavix) and aspirin for 30 days and then aspirin alone C) Clopidogrel alone for 1 year and then aspirin alone D) Clopidogrel and aspirin for 1 year and then aspirin alone E) Prasugrel (Effient) for 1 year with no anticoagulation after that

ANSWER: D Coronary artery stenting is a common procedure, and stent restenosis carries a high mortality rate. Current American College of Cardiology guidelines recommend dual antiplatelet therapy (aspirin with a second agent such as clopidogrel) for at least 12 months following the placement of a drug-eluting stent. Dual antiplatelet therapy with aspirin plus clopidogrel for more than 1 year gives no additional benefit and carries an additional risk of bleeding. Aspirin has been shown to be effective for the secondary prevention of heart disease and should be continued after 1 year.

A 25-year-old gravida 1 para 0 at 24 weeks gestation comes to your office with right lower extremity swelling and pain. Her pregnancy has been uncomplicated so far and her only medication is a prenatal vitamin. She does not have chest pain, shortness of breath, or fever. She recently started feeling the baby move, and an anatomy scan at 20 weeks gestation was normal. Lower extremity Doppler ultrasonography confirms a right lower extremity deep vein thrombosis (DVT). Laboratory studies including a CBC, coagulation studies, and renal function are normal. Which one of the following would be the most appropriate initial treatment of her DVT? A) Oral apixaban (Eliquis) B) Oral aspirin C) Oral warfarin (Coumadin) D) Subcutaneous enoxaparin (Lovenox) E) Subcutaneous heparin

ANSWER: D Enoxaparin is the most appropriate pharmacologic therapy for anticoagulation in patients who are pregnant. Aspirin is not used as treatment for deep vein thrombosis. Apixaban, warfarin, and heparin either have not been studied for use in pregnancy or there is data indicating potential fetal harm.

According to the recommendations of the American Heart Association, which one of the following patients requires endocarditis prophylaxis? A) A 10-year-old female with a previous history of Kawasaki disease without valvular dysfunction B) A 22-year-old female who underwent surgical repair of a ventricular septal defect 1 year ago C) A 28-year-old female with mitral valve prolapse without regurgitation D) A 35-year-old female with a history of infectious endocarditis in her 20s that was related to intravenous drug use E) A 42-year-old female with a history of rheumatic fever with chorea who has normal cardiovascular findings

ANSWER: D The American Heart Association and the American College of Cardiology have decreased the number of indications for antibiotic prophylaxis prior to dental procedures. Currently antibiotics are indicated for prosthetic cardiac valves, previous infective endocarditis, unrepaired cyanotic congenital heart disease or a repaired congenital defect with a residual shunt, and a cardiac transplant with valve regurgitation due to a structurally abnormal valve. Amoxicillin, 2 g, is the antibiotic prophylaxis of choice.

A 71-year-old male is hospitalized for community-acquired pneumonia. He has a past medical history of hypertension and a small, stable abdominal aortic aneurysm. Which one of the following antibiotics is likely to increase the risk of rupture of this patient's aneurysm? A) Aztreonam (Azactam) B) Ceftriaxone C) Doxycycline D) Levofloxacin

ANSWER: D The FDA issued a warning that systemic fluoroquinolones can increase the occurrence of aortic dissections or ruptures. Drugs in this group should be avoided in patients with an existing aortic aneurysm or in patients at increased risk for developing an aortic aneurysm unless there are no other treatment options available. Patients at increased risk include those with peripheral vascular disease, hypertension, Marfan syndrome, or Ehlers-Danlos syndrome. Similarly, the use of systemic fluoroquinolones should be avoided in the elderly. Aztreonam, ceftriaxone, and doxycycline are not associated with this side effect (SOR A).

Which one of the following is the greatest risk factor for abdominal aortic aneurysm (AAA)? A) Male sex B) Female sex C) White race D) A long duration of smoking E) Having a first degree relative with an AAA

ANSWER: D The following factors have been found to increase the risk for developing an abdominal aortic aneurysm (AAA): a history of smoking, advanced age, above-average height, having a first degree relative with an AAA, a personal history of atherosclerosis, high cholesterol levels, and hypertension. Smokers have a seven times greater risk of developing an AAA compared with nonsmokers. This single factor outweighs all of the other risk factors except age. Although women are less likely to develop an AAA, they have a 2-3 times greater chance of an AAA rupturing if it is present. In addition to significantly increasing the risk for AAA development, current smoking increases the risk for further AAA expansion and rupture. Epidemiologic studies suggest that the duration of smoking influences the risk for AAA significantly more than the total number of cigarettes smoked. The U.S. Preventive Services Task Force currently recommends one-time screening for AAA in males between the ages of 65 and 75 who have ever smoked (B recommendation). There was not enough evidence to determine the risk and benefits of screening females with the same risk factors (I recommendation).

A 50-year-old female sees you for follow-up of her hypertension. At her last visit 4 weeks ago you started her on lisinopril (Prinivil, Zestril), 10 mg daily, because of a blood pressure of 158/92 mm Hg and confirmed hypertension on ambulatory blood pressure monitoring. She is tolerating the medication well and has no side effects. She does not take any other medications. Today her blood pressure is 149/90 mm Hg, which you confirm on repeat measurement. This is also consistent with her home measurements. At her last visit a basic metabolic panel was normal. You repeat a basic metabolic panel today and the results are normal except for a BUN of 25 mg/dL (N 8-23) and a creatinine level of 1.5 mg/dL (N 0.6-1.1). At her last visit her BUN was 12 mg/dL and her creatinine level was 0.7 mg/dL. Which one of the following would be most appropriate at this time? A) Continue her current treatment regimen B) Increase lisinopril to 20 mg daily C) Continue lisinopril at the current dosage and add amlodipine (Norvasc), 5 mg daily D) Discontinue lisinopril and begin amlodipine, 5 mg daily E) Discontinue lisinopril and begin losartan (Cozaar), 25 mg daily

ANSWER: D This patient has essential hypertension and her goal blood pressure is <140/90 mm Hg based on JNC 8 guidelines, or 130/80 mm Hg based on the more recent recommendations of the American College of Cardiology/American Heart Association Task Force on Clinical Practice Guidelines. Until recently, it was recommended that physicians should tolerate a rise of <30% in serum creatinine after ACE inhibitor or angiotensin receptor blocker (ARB) initiation. Rises in serum creatinine of >30% from baseline increase the risk of renal failure, adverse cardiac outcomes, and death. A recent study suggests that rises in serum creatinine of <30% also put patients at risk for these outcomes, with a dose-response relationship between the magnitude of creatinine change and the risk of adverse outcomes. This patient has more than a 30% rise in creatinine and has no other factors, such as diabetes mellitus, heart failure, or chronic kidney disease, that would indicate a need for ACE or ARB therapy for her hypertension. Discontinuing her ACE inhibitor and starting a medication from a different class is the most appropriate treatment at this time. Based on JNC 8 guidelines, additional options for blood pressure medications include thiazide diuretics and calcium channel blockers.

A 4-year-old female is brought to your office because of a history of constipation over the past several months. Her mother reports that the child has 1-2 bowel movements per week composed of small lumps of hard stool. She strains to have the bowel movements, and they are painful. The child eats normally like her two siblings. Which one of the following would be most effective at this time? A) Daily fiber supplements B) Lactulose C) Magnesium hydroxide (Milk of Magnesia) D) Polyethylene glycol (MiraLAX) E) Senna

ANSWER: D This patient presents with symptoms compatible with functional constipation. Daily use of polyethylene glycol (PEG) solution has been found to be more effective than lactulose, senna, or magnesium hydroxide in head-to-head studies. Evidence does not support the use of fiber supplements in the treatment of functional constipation. No adverse effects were reported with PEG therapy at any dosing regimen. Low-dose regimens of PEG are 0.3 g/kg/day and high-dose regimens are up to 1.0-1.5 g/kg/day.

A 57-year-old male with diabetes mellitus and hypertension presents with a 1-month history of pain in his hands and elbows. His hands are shown below. On examination they are tender and he has soft swelling of the wrists, metacarpophalangeal (MCP) joints, and proximal interphalangeal (PIP) joints. Plain films show mild, diffuse bony erosions in the MCP and PIP joints. Which one of the following is the most likely diagnosis? A) Dermatomyositis B) Osteoarthritis C) Psoriatic arthritis D) Rheumatoid arthritis E) Systemic lupus erythematosus

ANSWER: D This patient's clinical findings and radiographs indicate a diagnosis of inflammatory arthritis, most likely rheumatoid arthritis. Symmetric small-joint inflammatory arthritis is typical of rheumatoid arthritis and systemic lupus erythematosus (SLE), but bony erosions are not seen in SLE. Psoriatic arthritis can also affect small joints but is typically not symmetric. Dermatomyositis can present with a thick, bright red rash over the metacarpophalangeal (MCP) and interphalangeal joints (Gottron's sign) but is typically associated with proximal muscle weakness rather than joint pain or erosions that can be seen on radiographs. Osteoarthritis does not typically cause the soft-tissue swelling seen in the image. It usually affects the distal and proximal interphalangeal joints while sparing the MCP joints, and it results in osteophytes and joint space narrowing that can be seen on radiographs.

A 38-year-old female has a 4-year history of intermittent abdominal pain occurring several days per week. She went to an urgent care clinic over the weekend and has brought her laboratory results for you to review. A comprehensive metabolic panel, CBC, and TSH level are all normal. She states that her stools have become harder and difficult to pass, with frequent straining and pain. She reports now having a bowel movement only 2-3 times a week. She tells you that she tried psyllium but it resulted in abdominal bloating. She is concerned because she has been missing work because of her symptoms. Her past and family medical histories are otherwise negative. Her vital signs are normal and her weight is unchanged from a previous wellness visit 6 months ago. An abdominal examination is notable for generalized pain with deep palpation. She does not have any masses, hepatosplenomegaly, or rebound tenderness. A rectal examination is also normal. Which one of the following is the most likely diagnosis? A) Celiac disease B) Colon cancer C) Hyperparathyroidism D) Inflammatory bowel disease E) Irritable bowel syndrome

ANSWER: E The Rome IV criteria for irritable bowel syndrome are symptom-based diagnostic criteria used to identify patients with irritable bowel syndrome (IBS). According to these criteria, IBS is defined as recurrent abdominal pain at least 1 day per week for at least 3 months. At least two of the following must be present: abdominal pain related to defecation, a change in stool form, and/or a change in stool frequency. The diagnosis can be made from the patient history, a physical examination, and a minimal laboratory evaluation. This patient meets the criteria.

A 64-year-old male with midsternal chest pain is brought to the emergency department by ambulance. He is on oxygen and an intravenous line is in place. Shortly after arrival he loses consciousness and becomes pulseless and apneic, and CPR is begun. Cardiac monitoring shows ventricular tachycardia with a rate of 160 beats/min. Which one of the following would be most appropriate at this point? A) Amiodarone, intravenous infusion, followed by synchronized cardioversion B) Adenosine (Adenocard), rapid intravenous push, repeated in 1-2 minutes if needed C) Epinephrine, intravenous push, followed by synchronized cardioversion D) Lidocaine (Xylocaine), intravenous push, repeated in 5 minutes if needed E) Defibrillation

ANSWER: E Pulseless ventricular tachycardia (VT) should be treated the same as ventricular fibrillation. The first step is defibrillation. If that is unsuccessful, epinephrine is administered and defibrillation is reattempted. Lidocaine, adenosine, and procainamide may be used for the initial treatment of a wide-complex tachycardia of uncertain type, but should not be used for the initial treatment of pulseless VT. Synchronized cardioversion alone would be indicated for the initial treatment of rapid unstable tachycardia with a pulse.

A 25-year-old landscaper presents with a 1-day history of pain and swelling in his left hand. Three days ago he sustained a puncture wound in the palm of the hand when he was stuck by a large thorn. He has generalized achiness and chills in addition to the hand pain. He has no significant past medical history. On examination the patient's temperature is 37.9°C (100.2°F). His left third finger is diffusely swollen, erythematous, and held in flexion. There is tenderness along the third tendon in the palm. A radiograph of the hand is negative for a fracture or foreign body. Which one of the following would be the most appropriate next step? A) Amoxicillin/clavulanate (Augmentin), 875/125 mg twice daily for 10 days B) Incision and drainage in the office with a culture of the wound C) A hand splint and nonurgent referral to an orthopedic surgeon D) Urgent MRI of the hand E) Urgent surgical consultation

ANSWER: E Pyogenic flexor tenosynovitis usually develops 2-5 days after a penetrating hand injury. The flexor tendon sheath has a poor vascular supply and the synovial fluid is a prime growth medium for bacteria. Flexor tenosynovitis is a clinical diagnosis characterized by the four "Kanavel" signs: pain with passive extension, tenderness with palpation of the tendon sheath, flexed position of the involved finger, and fusiform swelling of the finger. Treatment includes prompt intravenous antibiotics and surgical debridement and irrigation. Flexor tenosynovitis requires urgent surgical consultation and treatment. Patients with suspected flexor tenosynovitis should be seen by a surgeon within 72 hours of symptom onset (SOR C). Oral antibiotics and splinting of the hand alone are insufficient treatments for the condition. Incision and drainage would also not be sufficient to clear the infection. Ordering MRI can unnecessarily delay surgical consultation, although the surgeon may obtain one to guide treatment.

A 17-year-old female presents for acute care after tripping and falling on her right knee when stepping off her electric scooter. An examination does not reveal gross deformity or notable effusion. Range of motion in the right knee is limited to 100° of flexion. There is tenderness over the proximal tibia. She can only take two steps before being unable to bear weight on the knee due to pain. Which one of the following findings on the patient's history and examination should prompt you to order immediate knee radiographs? A) Her age B) Her sex C) Bony tenderness over the proximal tibia D) Limited range of motion E) The inability to take more than two steps

ANSWER: E Several decision support tools can help guide the decision to order imaging of an injured knee, such as the Ottawa Knee Rule, the Pittsburgh Knee Rule, and American College of Radiology (ACR) criteria. The inability to take four or more steps immediately after an injury or in the emergency setting is an indication for radiography in all three rules. Age is an indication for radiography in acute knee pain in patients over 55 years of age according to the Ottawa rule, or under 12 or over 50 years of age according to the Pittsburgh rule. The patient's sex does not factor into the criteria for imaging. Bony tenderness is an indication for imaging according to the ACR and Ottawa rules, but only if isolated over the proximal fibula or over the patella without other bony tenderness. The inability to flex the knee to 90° is also an indication for imaging according to the ACR and Ottawa rules.

A 68-year-old female presents for evaluation of shortness of breath with activity for the past several weeks. She used to walk 2 miles daily for exercise but can no longer do so because of dyspnea and chest tightness. She also reports mild lower extremity edema. She has a history of a bicuspid aortic valve and aortic stenosis. Echocardiography 1 year ago showed moderately severe aortic stenosis with a mean valve area of 1.1 cm2 . Echocardiography today shows aortic stenosis with an aortic valve area of 0.9 cm2 , a mean pressure gradient of 42 mm Hg, and a transaortic velocity of 4.3 m/sec. The ejection fraction is estimated to be 50%. Which one of the following is indicated at this time? A) Atorvastatin (Lipitor) B) Furosemide (Lasix) C) Lisinopril (Prinivil, Zestril) D) Metoprolol succinate (Toprol-XL) E) Referral for aortic valve replacement

ANSWER: E This patient has severe symptomatic aortic stenosis. The only therapy shown to improve symptoms and mortality in such patients is an aortic valve replacement. In patients with asymptomatic disease, watchful waiting is usually the recommended course of action. No medications or other therapies have been shown to prevent disease progression or alleviate symptoms. Patients with coexisting hypertension should be managed medically according to accepted guidelines. Diuretics should be used with caution due to their potential to reduce left ventricular filling and cardiac output, which leads to an increase in symptoms.

The physical examination is notable only for a BMI of 36.0 kg/m2 . Laboratory findings are notable for significant hyperlipidemia and you recommend starting a statin. She reports that she will undergo an elective total knee replacement next month and asks about the safety of starting a new medication before this surgery. You recommend that she A) start a statin immediately to decrease her risk of cardiovascular disease and perioperative mortality B) start a statin immediately to decrease her risk of cardiovascular disease, although her risk of perioperative mortality will not be affected C) start a statin immediately to decrease her risk of cardiovascular disease, stop the statin 1 week before surgery, and resume taking it after the surgery, to decrease her risk of perioperative mortality D) start a statin immediately after the surgery to decrease her risk of cardiovascular disease and perioperative mortality E) start a statin after she is released postoperatively by her surgeon to decrease her risk of cardiovascular disease and perioperative mortality

Answer A: Family physicians are often consulted for perioperative medical management. Studies have shown decreased perioperative mortality in patients who continue statins and in patients with clinical indications for statin therapy who start statins prior to undergoing vascular or high-risk surgeries such as joint replacement. A meta-analysis of 223,000 patients showed a significant reduction in perioperative mortality in patients receiving statin therapy versus placebo who underwent noncardiac surgical procedures. This patient has a clinical indication (multiple risk factors) to start statin therapy now.

A 70-year-old male presents to your office for follow-up after he was hospitalized for acute coronary syndrome. He has not experienced any pain since discharge and is currently in a supervised cardiac rehabilitation exercise program. His medications include aspirin, lisinopril (Prinivil, Zestril), and metoprolol, but he was unable to tolerate atorvastatin (Lipitor), 40 mg daily, because he developed muscle aches. Which one of the following would you recommend? A) Evolocumab (Repatha) B) Ezetimibe/simvastatin (Vytorin) C) Fenofibrate (Tricor) D) Niacin E) Omega-3 fatty acid supplements

Answer B - High-intensity statin therapy is recommended for patients younger than 75 years of age with known coronary artery disease. - For those who are intolerant of high-intensity statins, a trial of a moderate-intensity statin is appropriate. - There is evidence to support ezetimibe plus a statin in patients with acute coronary syndrome or chronic kidney disease. - Omega-3 fatty acids, fibrates, and niacin should not be prescribed for primary or secondary prevention of atherosclerotic cardiovascular disease because they do not affect patient-oriented outcomes. - PCSK9 inhibitors such as evolocumab are injectable monoclonal antibodies that lower LDL-cholesterol levels significantly and have produced some promising results, but more studies are needed to determine when this would be cost effective.

8. A 67-year-old female with hypertension and atrial fibrillation has been taking warfarin (Coumadin) for the past 10 years. She has been hemodynamically stable for many years with no complications from her atrial fibrillation. She is scheduled to undergo elective bladder sling surgery for urinary incontinence. She does not have any other significant past medical history. Which one of the following would be the most appropriate perioperative management of her warfarin. Which one of the following would be the most appropriate perioperative management of her warfarin? A) Continue warfarin without interruption B) Discontinue warfarin the day prior to surgery and provide bridge therapy with low molecular weight heparin C) Discontinue warfarin 2 days prior to surgery and restart it 2 days postoperatively unless there is a bleeding complication D) Discontinue warfarin 2 days prior to surgery and restart it 5 days postoperatively unless there is a bleeding complication E) Discontinue warfarin 5 days prior to surgery and restart it 12-24 hours postoperatively unless there is a bleeding complication

Answer E: Perioperative management of chronic anticoagulation requires an assessment of the patient's risk for thromboembolism and the risk of bleeding from the surgical procedure. High-risk patients include those with mechanical heart valves, a stroke or TIA within the past 3 months, venous thromboembolism within the past 3 months, or coronary stenting within the previous 12 months. High-risk patients require bridging therapy with low molecular weight heparin, while patients at low risk do not require bridging anticoagulation. For low-risk patients, it is recommended that warfarin be discontinued 5 days prior to surgery and restarted 12-24 hours postoperatively. This patient is at low risk for thromboembolism because her CHA2DS2-VASc score is 3. A patient with atrial fibrillation should receive bridging therapy with a CHA2DS2-VASc score 6. This patient's surgery is associated with a high risk for bleeding, so it is preferable to stop her warfarin 5 days before the operation.

A 90-year-old male presents to the emergency department with chest pain, dyspnea, and diaphoresis. He has experienced these symptoms intermittently since his wife died last week. An EKG shows ST elevation in the anterior leads, and cardiac enzymes are elevated. An echocardiogram shows apical ballooning of the left ventricle. Cardiac catheterization does not reveal coronary vascular disease. You plan to discharge the patient after observation overnight. Which one of the following would be the most appropriate management of this patient's stress-induced (Takotsubo) cardiomyopathy after discharge? A) Home medications only B) A cardiac event monitor to detect any rhythm abnormalities C) A diuretic, ACE inhibitor, and -blocker until his symptoms and the abnormalities seen on the echocardiogram resolve D) A statin, diuretic, ACE inhibitor, and -blocker to be continued indefinitely E) Pacemaker placement

Answer is C - Takotsubo cardiomyopathy, also known as stress-induced cardiomyopathy, can develop following emotional distress and is characterized by the abrupt onset of dysfunction of the left ventricle. - The clinical presentation and laboratory studies can mirror acute coronary syndrome and should be treated similarly. -Once symptoms and cardiac abnormalities resolve, treatment is no longer indicated and may be withdrawn if there are no signs of coronary disease. - Because this patient currently has cardiomyopathic abnormalities, a diuretic, ACE inhibitor, and -blocker are indicated.

Endocrine system

Answers

Gastrointestinal Qtsns 6/16

Answers

Three weeks after he had knee surgery, a 64-year-old male presents for follow-up of an emergency department visit for a pulmonary embolism. He has no previous history of pulmonary embolism and is otherwise in good health. He is being treated with apixaban (Eliquis). The recommended duration of anticoagulation therapy for this patient is: A) 1 month B) 3 months C) 6 months D) 9 months E) 12 months

B) The risk for VTE recurrence is dependent on patient factors, such as active cancers and thrombophilia. Current guidelines recommend treatment for at least 3 months. In patients who have a reversible provoking factor such as surgery, anticoagulation beyond 3 months is not recommended.

asthma diagnosis

Bronchoconstriction (defined as a reduction in FEV1 >/= 20%) observed at low levels of methacholine administration (<4 mg/mL) is consistent with asthma. If the FEV1/FVC ratio is reduced on initial spirometry, a bronchodilator response should be tested. A fixed or partially reversible obstructive pattern suggests an alternative diagnosis such as COPD, and full reversal after bronchodilator use is consistent with asthma. Inhaled corticosteroids are not appropriate for intermittent asthma

A 38-year-old female with a 6-month history of mild shortness of breath associated with some intermittent wheezing during upper respiratory infections presents for follow-up. You previously prescribed albuterol (Proventil, Ventolin) via metered-dose inhaler, which she says helps her symptoms. You suspect asthma. Pulmonary function testing reveals a normal FEV1/FVC ratio for her age. Which one of the following would be the most appropriate next step? A) Consider an alternative diagnosis B) Assess her bronchodilator response C) Perform a methacholine challenge D) Prescribe an inhaled corticosteroid E) Proceed with treatment for COPD

C Bronchial provocation testing is useful for ruling out asthma but less useful for making the diagnosis, and it should be limited to diagnosing asthma in athletes or in patients with symptoms despite normal spirometry findings or when spirometry is unavailable. Although the presence of atopy increases the likelihood of allergic asthma, its absence does not rule out asthma. In patients taking controller treatment, it may be necessary to step down the dose to confirm an asthma diagnosis. Patients should be advised to reduce their inhaled corticosteroid (ICS) dose by 25% to 50% or stop any other long-acting medication.

5. Among patients prescribed metered-dose inhaler treatments, technical errors in using the device are most likely in patients A) 15-30 years of age B) 30-60 years of age C) 60-75 years of age D) >75 years of age

D Studies have documented an increased rate of metered-dose inhaler (MDI) use errors in certain patient subgroups that could adversely affect the efficacy of treatment. For patients over the age of 15, the ability to demonstrate correct MDI use drops significantly with increasing age. One large study of MDI technique, after a minimum of 3 months of prescribed use, confirmed error frequencies of 61% in patients 15-30 years of age, 70% in patients 30-60 years of age, 77% in patients 60-75 years of age, and 86% in patients 75 years of age. Other patient characteristics also affect MDI error rates. A higher level of education and a diagnosis of asthma rather than COPD are both associated with fewer errors, whereas the frequency of errors is higher for females and lower-income patients. Disease severity and the presence of comorbidities has also been found to affect MDI use. The error rate is lower for patients who have had prior training, and the rate of proper usage relates directly to the duration of the training session.

right lateral ankle pain after twisting his ankle in a basketball game earlier today. The area is mildly swollen. A radiograph shows soft-tissue edema but no evidence of fracture.

Early mobilization leads to better outcomes with ankle sprains, and using a functional ankle brace such as a semi-rigid air stirrup brace or a soft lace-up ankle brace will protect the ankle from inversion and eversion sprains while still allowing for mobility during physical activity. These braces lead to improved functional outcomes at 1 month when compared to elastic compression taping. Patellar tendon straps relieve the pain associated with patellar tendinopathy but are not indicated with acute ankle sprains. Use of a semi-rigid air stirrup splint has been shown to be superior to neuromuscular training for prevention of recurrent ankle sprains.

flexible metatarsus adductus, the most common congenital foot deformity.

Flexible metatarsus adductus usually resolves spontaneously by 1 year of age and does not require treatment. Rigid metatarsus adductus should be treated with serial casting

COPD - GOLD Criteria

GOLD Criteria Fev1/fvc <70% for all below •Mild o FEV1 >80% o SABA •Moderate o FEV1 50-80% o LABA, anticholinergic •Severe o FEV1 30-50% o Send to pulmonary rehab o Acute exacerbation add steroids •Very Severe o FEV1 < 30% o Add oxygen

A 25-year-old female has a heart murmur on her postpartum visit. This was first noted at the age of 20. She has been asymptomatic. The murmur is systolic and increases in intensity with Valsalva maneuvers. Further questioning reveals that her two sisters died suddenly from cardiac problems in their early twenties. This patient should be evaluated for A) dilated cardiomyopathy B) hypertrophic cardiomyopathy C) peripartum cardiomyopathy D) restrictive cardiomyopathy

Hypertrophic cardiomyopathy is the most common type of cardiomyopathy, with a prevalence of 1:500. It involves left ventricular hypertrophy without chamber dilatation. It is caused by autosomal dominant genetic mutations and is associated with sudden death. Dilated cardiomyopathy is a leading cause of heart failure but most patients are symptomatic. Peripartum cardiomyopathy may occur during and after pregnancy and presents as heart failure. Restrictive cardiomyopathy presents with right-sided heart failure.

A 2-year-old female is brought to your office for a well child check. She had an episode of coughing and mild bronchospasm 3 months ago that was successfully treated with albuterol (Proventil, Ventolin). The mother asks you if there are any factors that would increase the patient's risk of asthma. Which one of the following factors would increase this patient's risk of asthma? A) Living in a high microbial environment B) Exposure to respiratory syncytial virus C) Recurrent otitis media D) Persistent lactose intolerance

Immunologic profiles of patients with asthma are influenced by environmental exposures. Those who are exposed to respiratory syncytial virus as an infant have an increased risk, whereas those who are exposed to a high microbial environment have a lower risk than those without such exposure. Otitis media and lactose intolerance are not known to be associated with asthma risk.

A 75-year-old male nursing home resident is brought to the emergency department with a cough and fever. His past medical history is significant for coronary artery disease, COPD, hypertension, and osteoarthritis. On examination he has a blood pressure of 145/90 mm Hg, a pulse rate of 84 beats/min, and an oxygen saturation of 89% on room air. A physical examination is remarkable for mildly labored breathing and crackles in his left lower lung field. A chest radiograph confirms left lower lobe pneumonia. He is admitted to the hospital for intravenous antibiotics. Which one of the following would be the most appropriate antibiotic treatment? A) Cefdinir only B) Piperacillin/tazobactam (Zosyn) only C) Ceftriaxone and azithromycin (Zithromax) D) Cefixime (Suprax) and vancomycin (Vancocin) E) Piperacillin/tazobactam, vancomycin, and ciprofloxacin (Cipro)

Inpatient, No ICU. Hospitalized patients who are not admitted to the ICU should receive a respiratory fluoroquinolone or a beta-lactam antibiotic and a macrolide. Observational data suggest lower short-term mortality when antibiotic therapy is administered within four to eight hours of hospital arrival in patients with moderate or severe pneumonia.36 Table 7 provides recommended criteria for transitioning patients from intravenous to oral antibiotics.36 Although oral doxycycline is not included in existing guidelines, observational studies show that it has similar effectiveness as levofloxacin (Levaquin) alone or in combination with ceftriaxone in the inpatient, non-ICU setting, and is associated with fewer Clostridium difficile infections.39,40,43,44 Trials of a new cephalosporin (ceftaroline [Teflaro]) show promising results, although treatment should be reserved for patients with highly resistant gram-positive organisms.

kNOWLEDGE OF ENDEMIC FUNGI

Knowledge of endemic fungi capable of causing infection in otherwise healthy patients can be very helpful in ensuring an appropriate evaluation. Coccidioidomycosis is a common infection in the southwestern United States. In addition to the symptoms in this patient, coccidioidomycosis can also present with a rash such as erythema nodosum. Histoplasmosis is most common in the Midwest and with low-level exposure symptoms are usually mild or absent. Blastomycosis is also present in the Midwest, as well as in the Atlantic and southeastern states. Symptoms include an abrupt onset of fever, chills, pleuritic chest pain, arthralgias, and myalgias. The cough is initially nonproductive but frequently becomes purulent. Cryptococcosis and mucormycosis are more opportunistic infections occurring in immunocompromised hosts.

Pharmacotherapy associated with osteoporosis

Medications reported to be associated with osteoporosis and increased fracture risk include antiepileptic drugs, long-term heparin, cyclosporine, tacrolimus, aromatase inhibitors, glucocorticoids, gonadotropin-releasing hormone agonists, thiazolidinediones, excessive doses of levothyroxine, proton pump inhibitors, SSRIs, parenteral nutrients, medroxyprogesterone contraceptives, methotrexate, and aluminum antacids.

49. A 67-year-old male diagnosed with polymyalgia rheumatica is started on long-term prednisone therapy. Which one of the following is the recommended first-line agent to prevent steroid-induced osteoporosis? A) Alendronate (Fosamax) B) High-dose vitamin D C) Raloxifene (Evista) D) Teriparatide (Forteo)

Oral bisphosphonates are recommended as first-line agents for preventing glucocorticoid-induced osteoporotic fractures, although intravenous bisphosphonates can be used if patients are unable to use the oral forms. Supplementation of calcium (800-1000 mg) and vitamin D (400-800 IU) is also recommended. Raloxifene and teriparatide are options when bisphosphonate therapy fails or is contraindicated (SOR A).

A 55-year-old female presents with the new onset of palpitations. An underlying cardiac cause should be suspected if the patient's palpitations: A) affect her sleep B) are associated with dry mouth C) are worse in public places D) last less than 5 minutes

Palpitations are a common symptom in ambulatory care. Cardiac causes are the most worrisome so it is important to distinguish cardiac from noncardiac causes. Patients with a history of cardiovascular disease, palpitations that affect their sleep, or palpitations that occur at work have an increased risk of an underlying cardiac cause (positive likelihood ratio 2.0-2.3) (SOR C). Psychiatric illness, adverse effects of medications, and substance abuse are other common causes. Palpitations that are worse in public places and those of very short duration (<5 minutes), especially if there is a history of anxiety, are often related to panic disorder. However, even a known behavioral issue should not be presumed to be the cause of palpitations, as nonpsychiatric causes are found in up to 13% of such cases. The use of illicit substances such as cocaine and methamphetamine can cause palpitations that are associated with dry mouth, pupillary dilation, sweating, and aberrant behavior. Excessive caffeine can also cause palpitations.

A 66-year-old male recently underwent percutaneous angioplasty for persistent angina with exertion. He does not have any symptoms now. His LDL-cholesterol level is 90 mg/dL. Which one of the following would be most appropriate for secondary prevention of this patient's coronary artery disease? A) No drug treatment B) Evolocumab (Repatha), 140 mg subcutaneously every 2 weeks C) Ezetimibe (Zetia), 10 mg daily D) Rosuvastatin (Crestor), 20 mg daily E) Simvastatin (Zocor), 40 mg daily

Patients <75 years of age with established coronary artery disease should be on high-intensity statin regimens if tolerated. These regimens include atorvastatin, 40-80 mg/day, and rosuvastatin, 20-40 mg/day. Moderate-intensity regimens include simvastatin, 40 mg/day. Monotherapy with non-statin medications (bile acid sequestrants, niacin, ezetimibe, and fibrates) does not reduce cardiovascular morbidity or mortality. The PCSK9 inhibitors evolocumab and alirocumab are second-line or add-on therapies at this time.

Charcot Neuroarthropathy (HY+)

Presents as a red, hot swollen foot. It is bone and joint changes that occur secondary to *loss of sensation* and accompanying a variety of disorders. To differentiate from arthritis, order a triphasic bone scan (WBC label). This is a life-changing event as it requires a special boot and may take months to years to heal.

This patient has Dupuytren's disease with a contracture of the affected finger.

Surgical release is indicated when the metacarpophalangeal joint contracture reaches 30° or with any degree of contracture of the proximal interphalangeal joint.

Charcot neuroarthropathy

THIS = noninfective bone & joint destruction that occurs in a well-perfused and insensate foot (commonly in diabetics)

Asthma Environmental risk factors

TRIGGERS AND ENVIRONMENTAL CONTROL Asthma triggers include allergens from dust mites or mold spores, animal dander, cockroaches, pollen, indoor and outdoor pollutants, irritants (e.g., tobacco smoke, smoke from wood-burning stoves or fireplaces, perfumes, cleaning agents), pharmacologic triggers (e.g., aspirin or other nonsteroidal anti-inflammatory drugs, beta blockers and sulfites), physical triggers (e.g., exercise, hyper-ventilation, cold air) and physiologic factors (e.g., stress, gastroesophageal reflux, respiratory infection [viral, bacterial] and rhinitis) Environmental control measures include removing carpets from the patient's bedroom and living areas, weekly washing of bedding and clothing in water hotter than 55° C (130°F), the use of specially designed mattress and pillow covers, removing stuffed animals and similar objects that are likely to harbor allergens, keeping pets outdoors and using special furnace filters to remove airborne allergens. The Web site of the American Academy of Allergy, Asthma and Immunology is an excellent source of valuable, scientifically based information and specialized products for persons with asthma. Up to 80 percent of asthmatic children have allergic rhinitis. If specific IgE hypersensitivity has been identified by radioallergosorbent test (RAST) or skin testing, the triggers to be avoided can be specified.21 Consultation with an allergist can define the optimal regimen to reduce sensitivity to specific allergens. Because exposure to tobacco smoke is a major cause of respiratory problems in children who are predisposed to or already have asthma, exposure should be strictly avoided

Lung cancer screening

The 2013 U.S. Preventive Services Task Force lung cancer screening guidelines recommend annual low-dose CT screening for all adults between the ages of 55 and 80 who have a 30-pack-year smoking history and either currently smoke or have smoked within the past 15 years (B recommendation).

Chronic Rhinosinusitis (CRS)

The American Academy of Otolaryngology defines chronic rhinosinusitis as the presence of two of four cardinal symptoms, which include nasal drainage, nasal obstruction, facial pain or pressure, and hyposmia or anosmia, along with objective signs on examination or radiographic studies. This patient has three cardinal symptoms of chronic rhinosinusitis and objective evidence on the physical examination. Allergic rhinitis can be associated with chronic rhinosinusitis but would also present with allergic symptoms.

Screening for developmental dysplasia of the hip (DDH)

The American Academy of Pediatrics, however, recommends routine screening of all newborns with physical examination maneuvers, and targeted screening ultrasonography for infants who were breech in the third trimester, have a family history of DDH, or have a personal history of instability. Given this, decisions should be individualized. Additional risk factors include female sex, firstborn status, oligohydramnios, and a large-for-gestational-age infant.

Influenza vaccine

The CDC's Advisory Committee on Immunization Practices recommends that patients with egg allergy receive influenza vaccination. Previously unvaccinated patients ages 6 months to 8 years should receive two doses of either trivalent or quadrivalent vaccine separated by 1 month.

A previously healthy 45-year-old female presents with upper abdominal pain and dysphagia. An upper GI series reveals no significant reflux. On esophagogastroduodenoscopy the esophagus has a ringed appearance and a biopsy reveals >15 eosinophils/hpf. Helicobacter pylori testing is negative. She does not currently take any medications. Which one of the following would be the best initial treatment? A) Budesonide oral suspension, 1 mg twice daily B) Fexofenadine, 180 mg daily C) Pantoprazole (Protonix), 40 mg once daily D) Prednisone, 40 mg daily for 7 days E) Ranitidine (Zantac), 150 mg once daily

The clinical presentation and esophagogastroduodenoscopy findings indicate eosinophilic esophagitis (EoE) in this patient. In the absence of other causes of eosinophilia, the presence of >15 eosinophils/hpf is considered diagnostic. Application of corticosteroids to the esophagus is generally the treatment of choice, either in the form of an oral suspension of budesonide or an inhaled corticosteroid sprayed into the mouth and swallowed.

On esophagogastroduodenoscopy the esophagus has a ringed appearance and a biopsy reveals >15 eosinophils/hpf. Helicobacter pylori testing is negative. She does not currently take any medications. Which one of the following would be the best initial treatment? A) Budesonide oral suspension, 1 mg twice daily B) Fexofenadine, 180 mg daily C) Pantoprazole (Protonix), 40 mg once daily D) Prednisone, 40 mg daily for 7 days E) Ranitidine (Zantac), 150 mg once daily

The clinical presentation and esophagogastroduodenoscopy findings indicate eosinophilic esophagitis (EoE) in this patient. In the absence of other causes of eosinophilia, the presence of >15 eosinophils/hpf is considered diagnostic. Application of corticosteroids to the esophagus is generally the treatment of choice, either in the form of an oral suspension of budesonide or an inhaled corticosteroid sprayed into the mouth and swallowed.

Scaphoid fracture

The finding of anatomic snuffbox tenderness is highly sensitive but not specific for a scaphoid fracture. Initial radiographs often do not demonstrate a fracture. When there is a high clinical suspicion for a scaphoid fracture but radiographs are negative, it is reasonable to immobilize in a thumb spica splint and reevaluate in 2 weeks.

Chronic cough in adults

The most common causes of chronic cough in adults include upper airway cough syndrome, tobacco use, GERD, asthma, and ACE inhibitor use. The physical examination of this patient is most consistent with upper airway cough syndrome, previously referred to as postnasal drip syndrome.

The novel anticoagulants (NOACs) include apixaban (Eliquis), dabigatran (Pradaxa), edoxaban (Savaysa), and rivaroxaban (Xarelto). Which one of the following should be considered when starting or adjusting the dosage of a NOAC? A) Serum albumin B) INR C) Liver enzymes D) Partial thromboplastin time E) Renal function

The novel anticoagulants (NOACs) require dosage adjustments based on renal function. There are no dosing recommendations for NOACs based on liver function or albumin level. The INR is used to adjust warfarin dosing and the partial thromboplastin time is used to adjust heparin dosing.

A 69-year-old female with hypertension, hyperlipidemia, and coronary artery disease had a myocardial infarction 1 year ago that was treated with percutaneous stenting. She was recently diagnosed with atrial fibrillation and takes diltiazem (Cardizem) for rate control. She is also taking lisinopril (Prinivil, Zestril), atorvastatin (Lipitor), atenolol (Tenormin), and aspirin, 81 mg. Of the following, which option would be best for thromboembolism prevention in this patient? A) Continue the current medication regimen B) Increase the aspirin dosage to 325 mg daily C) Discontinue aspirin and beg D) Add clopidogrel (Plavix)

This patient has a CHA2DS2-VASc score of 3 (hypertension, age 65-74, female), which classifies her as high risk for thromboembolism. Oral anticoagulation is indicated for patients with a score of 2 or more, who are at high risk for pulmonary embolism/deep vein thrombosis (PE/DVT) (SOR C). Patients with a score of 0-1 have a low to medium risk and may use aspirin with or without clopidogrel. In patients with atrial fibrillation and stable coronary artery disease, novel oral anticoagulants are preferred (SOR A). They reduce the risk of reinfarction, stroke, and overall mortality in patients with a past history of myocardial infarction, and also help prevent PE/DVT.

Charcot neuroarthropathy 1

This patient has acute Charcot neuroarthropathy, an inflammatory condition that occurs in obese patients with peripheral neuropathy and ultimately leads to foot deformities (the classic rocker-bottom foot) and resultant ulcerations and infections. Its clinical appearance can easily be initially mistaken for cellulitis. However, the absence of tenderness and other signs of infection such as fever, an elevated WBC count, and inflammatory markers is not consistent with cellulitis. Radiography is an appropriate initial imaging modality but the results are often interpreted as normal early in the disease process. MRI is the modality of choice for a definitive diagnosis and may demonstrate periarticular bone marrow edema, adjacent soft-tissue edema, joint effusion, and microtrabecular or stress fractures. The treatment of acute Charcot neuroarthropathy is immobilization with total contact casting, which increases the total surface area of contact to the entire lower extremity, distributing pressure away from the foot. Immobilization is typically required for at least 3-4 months but in some cases may be needed for up to 12 months.

sarcoidosis - EKG --> mgt

Type II second degree AV block

Lifestyle modifications addressing diet, physical activity, and weight are important in the treatment of hypertension, particularly for African-American and Hispanic patients.

When antihypertensive drugs are also required, the best options may vary according to the racial and ethnic background of the patient. The presence or absence of comorbid conditions is also important to consider. For African-Americans, thiazide diuretics and calcium channel blockers, both as monotherapy and as a component in multidrug regimens, have been shown to be more effective in lowering blood pressure than ACE inhibitors, angiotensin II receptor blockers, or -blockers, and should be considered as first-line options over the other classes of antihypertensive drugs unless a comorbid condition is present that would be better addressed with a different class of drugs.

Written Action Plans for Self-Management of COPD Exacerbations

Written self-management plans, especially those that include guidance on smoking cessation, increase health-related quality of life in patients with COPD. They also decrease the need for respiratory-related hospital admissions among patients with high or low baseline risk.1 (Strength of Recommendation: A, based on consistent, good-quality patient-oriented evidence.)

2018 set of questions

answer options for 2018

pulmonary circulation

flow of blood from the heart to the lungs and back to the heart

musculoskeletal system

the system of bones and skeletal muscles that support and protect the body and permit movement


Conjuntos de estudio relacionados

PSY 350 Michael Varnum ASU Exam 1 Review

View Set

Chapter 17 practice test biology

View Set

finance 310 multiple choice exam 1

View Set

Fundamentos de marketing: Cap 12

View Set

Тема 11. Система видатків бюджету та кошторисне фінансування бюджетних установ

View Set

Completion/Short Answer Chapter 1

View Set

Chapter 49: Assessment and Management of Patients With Hepatic Disorders

View Set